Difference between revisions of "2024 AMC 10B Problems/Problem 6"

(Solution 1)
 
(3 intermediate revisions by 2 users not shown)
Line 2: Line 2:
 
A rectangle has integer length sides and an area of 2024. What is the least possible perimeter of the rectangle?
 
A rectangle has integer length sides and an area of 2024. What is the least possible perimeter of the rectangle?
  
<math>\textbf{(A) } 160 \qquad\textbf{(B) } 15 \qquad\textbf{(C) } 16 \qquad\textbf{(D) } 17 \qquad\textbf{(E) } 18</math>
+
<math>\textbf{(A) } 160 \qquad\textbf{(B) } 180 \qquad\textbf{(C) } 16 \qquad\textbf{(D) } 17 \qquad\textbf{(E) } 18</math>
  
==Solution 1==
+
==Solution 1 - Prime Factorization==
  
B) 180, 2024=44*46, (44+46)*2=180.
+
We can start by assigning the values x and y for both sides. Here is the equation representing the area:
 +
 
 +
 
 +
<math>x \cdot y = 2024</math>
 +
 
 +
Let's write out 2024 fully factorized.
 +
 
 +
 
 +
<math>2^3 \cdot 11 \cdot 23</math>
 +
 
 +
Since we know that <math>x^2 > (x+1)(x-1)</math>, we want the two closest numbers possible. After some quick analysis, those two numbers are <math>44</math> and <math>46</math>. <math>\\44+46=90</math>
 +
 
 +
Now we multiply by <math>2</math> and get <math>\boxed{\textbf{(B) }180}.</math>
 +
 
 +
Solution by [[User:IshikaSaini|IshikaSaini]].
  
 
==See also==
 
==See also==
 
{{AMC10 box|year=2024|ab=B|num-b=5|num-a=7}}
 
{{AMC10 box|year=2024|ab=B|num-b=5|num-a=7}}
 
{{MAA Notice}}
 
{{MAA Notice}}

Latest revision as of 01:30, 14 November 2024

Problem

A rectangle has integer length sides and an area of 2024. What is the least possible perimeter of the rectangle?

$\textbf{(A) } 160 \qquad\textbf{(B) } 180 \qquad\textbf{(C) } 16 \qquad\textbf{(D) } 17 \qquad\textbf{(E) } 18$

Solution 1 - Prime Factorization

We can start by assigning the values x and y for both sides. Here is the equation representing the area:


$x \cdot y = 2024$

Let's write out 2024 fully factorized.


$2^3 \cdot 11 \cdot 23$

Since we know that $x^2 > (x+1)(x-1)$, we want the two closest numbers possible. After some quick analysis, those two numbers are $44$ and $46$. $\\44+46=90$

Now we multiply by $2$ and get $\boxed{\textbf{(B) }180}.$

Solution by IshikaSaini.

See also

2024 AMC 10B (ProblemsAnswer KeyResources)
Preceded by
Problem 5
Followed by
Problem 7
1 2 3 4 5 6 7 8 9 10 11 12 13 14 15 16 17 18 19 20 21 22 23 24 25
All AMC 10 Problems and Solutions

The problems on this page are copyrighted by the Mathematical Association of America's American Mathematics Competitions. AMC logo.png